Summer is a great time to explore cool problems to keep your skills sharp!  Schedule a class today!

G
Topic
First Poster
Last Poster
PAMO 2022 Problem 1 - Line Tangent to Circle Through Orthocenter
DylanN   5
N 11 minutes ago by Y77
Source: 2022 Pan-African Mathematics Olympiad Problem 1
Let $ABC$ be a triangle with $\angle ABC \neq 90^\circ$, and $AB$ its shortest side. Let $H$ be the orthocenter of $ABC$. Let $\Gamma$ be the circle with center $B$ and radius $BA$. Let $D$ be the second point where the line $CA$ meets $\Gamma$. Let $E$ be the second point where $\Gamma$ meets the circumcircle of the triangle $BCD$. Let $F$ be the intersection point of the lines $DE$ and $BH$.

Prove that the line $BD$ is tangent to the circumcircle of the triangle $DFH$.
5 replies
DylanN
Jun 25, 2022
Y77
11 minutes ago
Conditional geo with centroid
a_507_bc   6
N an hour ago by LeYohan
Source: Singapore Open MO Round 2 2023 P1
In a scalene triangle $ABC$ with centroid $G$ and circumcircle $\omega$ centred at $O$, the extension of $AG$ meets $\omega$ at $M$; lines $AB$ and $CM$ intersect at $P$; and lines $AC$ and $BM$ intersect at $Q$. Suppose the circumcentre $S$ of the triangle $APQ$ lies on $\omega$ and $A, O, S$ are collinear. Prove that $\angle AGO = 90^{o}$.
6 replies
a_507_bc
Jul 1, 2023
LeYohan
an hour ago
Channel name changed
Plane_geometry_youtuber   0
an hour ago
Hi,

Due to the search handle issue in youtube. My channel is renamed to Olympiad Geometry Club. And the new link is as following:

https://www.youtube.com/@OlympiadGeometryClub

Recently I introduced the concept of harmonic bundle. I will move on to the conjugate median soon. In the future, I will discuss more than a thousand theorems on plane geometry and hopefully it can help to the students preparing for the Olympiad competition.

Please share this to the people may need it.

Thank you!
0 replies
Plane_geometry_youtuber
an hour ago
0 replies
IMO Shortlist 2010 - Problem G1
Amir Hossein   134
N an hour ago by happypi31415
Let $ABC$ be an acute triangle with $D, E, F$ the feet of the altitudes lying on $BC, CA, AB$ respectively. One of the intersection points of the line $EF$ and the circumcircle is $P.$ The lines $BP$ and $DF$ meet at point $Q.$ Prove that $AP = AQ.$

Proposed by Christopher Bradley, United Kingdom
134 replies
Amir Hossein
Jul 17, 2011
happypi31415
an hour ago
Divisors on number
RagvaloD   34
N 2 hours ago by cubres
Source: All Russian Olympiad 2017,Day1,grade 10,P5
$n$ is composite. $1<a_1<a_2<...<a_k<n$ - all divisors of $n$. It is known, that $a_1+1,...,a_k+1$ are all divisors for some $m$ (except $1,m$). Find all such $n$.
34 replies
RagvaloD
May 3, 2017
cubres
2 hours ago
IMO ShortList 2002, number theory problem 2
orl   59
N 2 hours ago by cubres
Source: IMO ShortList 2002, number theory problem 2
Let $n\geq2$ be a positive integer, with divisors $1=d_1<d_2<\,\ldots<d_k=n$. Prove that $d_1d_2+d_2d_3+\,\ldots\,+d_{k-1}d_k$ is always less than $n^2$, and determine when it is a divisor of $n^2$.
59 replies
orl
Sep 28, 2004
cubres
2 hours ago
None of the circles contains the pentagon - ILL 1970, P34
Amir Hossein   1
N 2 hours ago by legogubbe
In connection with a convex pentagon $ABCDE$ we consider the set of ten circles, each of which contains three of the vertices of the pentagon on its circumference. Is it possible that none of these circles contains the pentagon? Prove your answer.
1 reply
Amir Hossein
May 21, 2011
legogubbe
2 hours ago
interesting incenter/tangent circle config
LeYohan   0
2 hours ago
Source: 2022 St. Mary's Canossian College F4 Final Exam Mathematics Paper 1, Q 18d of 18 (modified)
$BC$ is tangent to the circle $AFDE$ at $D$. $AB$ and $AC$ cut the circle at $F$ and $E$ respectively. $I$ is the in-centre of $\triangle ABC$, and $D$ is on the line $AI$. $CI$ and $DE$ intersect at $G$, while $BI$ and $FD$ intersect at $P$. Prove that the points $P, F, G, E$ lie on a circle.
0 replies
LeYohan
2 hours ago
0 replies
interesting geo config (2/3)
Royal_mhyasd   5
N 2 hours ago by Royal_mhyasd
Source: own
Let $\triangle ABC$ be an acute triangle and $H$ its orthocenter. Let $P$ be a point on the parallel through $A$ to $BC$ such that $\angle APH = |\angle ABC-\angle ACB|$. Define $Q$ and $R$ as points on the parallels through $B$ to $AC$ and through $C$ to $AB$ similarly. If $P,Q,R$ are positioned around the sides of $\triangle ABC$ as in the given configuration, prove that $P,Q,R$ are collinear.
5 replies
Royal_mhyasd
Yesterday at 11:36 PM
Royal_mhyasd
2 hours ago
interesting geometry config (3/3)
Royal_mhyasd   2
N 2 hours ago by Royal_mhyasd
Let $\triangle ABC$ be an acute triangle, $H$ its orthocenter and $E$ the center of its nine point circle. Let $P$ be a point on the parallel through $C$ to $AB$ such that $\angle CPH = |\angle BAC-\angle ABC|$ and $P$ and $A$ are on different sides of $BC$ and $Q$ a point on the parallel through $B$ to $AC$ such that $\angle BQH = |\angle BAC - \angle ACB|$ and $C$ and $Q$ are on different sides of $AB$. If $B'$ and $C'$ are the reflections of $H$ over $AC$ and $AB$ respectively, $S$ and $T$ are the intersections of $B'Q$ and $C'P$ respectively with the circumcircle of $\triangle ABC$, prove that the intersection of lines $CT$ and $BS$ lies on $HE$.

final problem for this "points on parallels forming strange angles with the orthocenter" config, for now. personally i think its pretty cool :D
2 replies
Royal_mhyasd
Today at 7:06 AM
Royal_mhyasd
2 hours ago
Convex Quadrilateral with Bisector Diagonal
matinyousefi   8
N 3 hours ago by lpieleanu
Source: Germany TST 2017
In a convex quadrilateral $ABCD$, $BD$ is the angle bisector of $\angle{ABC}$. The circumcircle of $ABC$ intersects $CD,AD$ in $P,Q$ respectively and the line through $D$ parallel to $AC$ cuts $AB,AC$ in $R,S$ respectively. Prove that point $P,Q,R,S$ lie on a circle.
8 replies
matinyousefi
Apr 11, 2020
lpieleanu
3 hours ago
Kids in clubs
atdaotlohbh   0
3 hours ago
There are $6k-3$ kids in a class. Is it true that for all positive integers $k$ it is possible to create several clubs each with 3 kids such that any pair of kids are both present in exactly one club?
0 replies
atdaotlohbh
3 hours ago
0 replies
Turbo's en route to visit each cell of the board
Lukaluce   22
N 3 hours ago by HamstPan38825
Source: EGMO 2025 P5
Let $n > 1$ be an integer. In a configuration of an $n \times n$ board, each of the $n^2$ cells contains an arrow, either pointing up, down, left, or right. Given a starting configuration, Turbo the snail starts in one of the cells of the board and travels from cell to cell. In each move, Turbo moves one square unit in the direction indicated by the arrow in her cell (possibly leaving the board). After each move, the arrows in all of the cells rotate $90^{\circ}$ counterclockwise. We call a cell good if, starting from that cell, Turbo visits each cell of the board exactly once, without leaving the board, and returns to her initial cell at the end. Determine, in terms of $n$, the maximum number of good cells over all possible starting configurations.

Proposed by Melek Güngör, Turkey
22 replies
Lukaluce
Apr 14, 2025
HamstPan38825
3 hours ago
n lamps
pohoatza   47
N 3 hours ago by yayyayyay
Source: IMO Shortlist 2006, Combinatorics 1, AIMO 2007, TST 2, P1
We have $ n \geq 2$ lamps $ L_{1}, . . . ,L_{n}$ in a row, each of them being either on or off. Every second we simultaneously modify the state of each lamp as follows: if the lamp $ L_{i}$ and its neighbours (only one neighbour for $ i = 1$ or $ i = n$, two neighbours for other $ i$) are in the same state, then $ L_{i}$ is switched off; – otherwise, $ L_{i}$ is switched on.
Initially all the lamps are off except the leftmost one which is on.

$ (a)$ Prove that there are infinitely many integers $ n$ for which all the lamps will eventually be off.
$ (b)$ Prove that there are infinitely many integers $ n$ for which the lamps will never be all off.
47 replies
pohoatza
Jun 28, 2007
yayyayyay
3 hours ago
Polygons Which Don't Fit
somebodyyouusedtoknow   1
N Apr 27, 2025 by kiyoras_2001
Source: San Diego Honors Math Contest 2025 Part II, Problem 1
Let $P_1,P_2,\ldots,P_n$ be polygons, no two of which are similar. Show that there are polygons $Q_1,Q_2,\ldots,Q_n$ where $Q_i$ is similar to $P_i$ so that for no $i \neq j$ does $Q_i$ contain a polygon that's congruent to $Q_j$.

Note. Here, the word "contain" means for the construction we have, we cannot select a size for $Q_j$ so that $Q_j$ is wholly contained in $Q_i$, and so it does not intersect the edges of $Q_i$ at all.
1 reply
somebodyyouusedtoknow
Apr 26, 2025
kiyoras_2001
Apr 27, 2025
Polygons Which Don't Fit
G H J
Source: San Diego Honors Math Contest 2025 Part II, Problem 1
The post below has been deleted. Click to close.
This post has been deleted. Click here to see post.
somebodyyouusedtoknow
259 posts
#1
Y by
Let $P_1,P_2,\ldots,P_n$ be polygons, no two of which are similar. Show that there are polygons $Q_1,Q_2,\ldots,Q_n$ where $Q_i$ is similar to $P_i$ so that for no $i \neq j$ does $Q_i$ contain a polygon that's congruent to $Q_j$.

Note. Here, the word "contain" means for the construction we have, we cannot select a size for $Q_j$ so that $Q_j$ is wholly contained in $Q_i$, and so it does not intersect the edges of $Q_i$ at all.
This post has been edited 1 time. Last edited by somebodyyouusedtoknow, Apr 26, 2025, 11:41 PM
Reason: Added clarity for the notes
Z K Y
The post below has been deleted. Click to close.
This post has been deleted. Click here to see post.
kiyoras_2001
678 posts
#2
Y by
Induct on $n$. The base case $n=2$ is obvious; for fixed $Q_1=P_1$ consider the largest copy of $P_2$ inside it and slightly enlarge it to obtain $Q_2$.

Now the step $n\to n+1$. That is, given $Q_1, \ldots, Q_n$ with $Q_i \not\subset Q_j$ for all $i\ne j$ we seek for appropriate $Q_{n+1}$.

For each $i\in [n]:= \{1, \ldots, n\}$ define $R_i^-$ as the largest copy of $P_{n+1}$ inside $Q_i$. As well as define $R_i^+$ as the smallest copy of $P_{n+1}$ containing $Q_i$.

We want to show that there is a copy of $P_{n+1}$ which is larger than $R_i^-$ and smaller than $R_i^+$ for all $i\in [n]$. That is, we need to prove that $\max_{i\in [n]} R_i^- \subsetneq \min_{i\in [n]}R_i^+$. Suppose not, then there are $i\ne j$ such that $Q_i \subseteq R_i^+ \subseteq R_j^- \subseteq Q_j$. This contradiction proves that we can select $Q_{n+1}$ appropriately.
Z K Y
N Quick Reply
G
H
=
a